Difference between revisions of "2017 AMC 10A Problems/Problem 2"

m
(Video Solution)
 
(One intermediate revision by one other user not shown)
Line 10: Line 10:
  
 
==Video Solution==
 
==Video Solution==
https://youtu.be/rxeqBnHDQvI
+
https://youtu.be/str7kmcRMY8
 +
 
 +
https://youtu.be/9VGPpemq-qg
 +
 
 +
~savannahsolver
  
 
==See Also==
 
==See Also==
 
{{AMC10 box|year=2017|ab=A|num-b=1|num-a=3}}
 
{{AMC10 box|year=2017|ab=A|num-b=1|num-a=3}}
 
{{MAA Notice}}
 
{{MAA Notice}}

Latest revision as of 17:10, 16 June 2020

Problem

Pablo buys popsicles for his friends. The store sells single popsicles for $$1$ each, $3$-popsicle boxes for $$2$ each, and $5$-popsicle boxes for $$3$. What is the greatest number of popsicles that Pablo can buy with $$8$?

$\textbf{(A)}\ 8\qquad\textbf{(B)}\ 11\qquad\textbf{(C)}\ 12\qquad\textbf{(D)}\ 13\qquad\textbf{(E)}\ 15$

Solution

$$3$ boxes give us the most popsicles/dollar, so we want to buy as many of those as possible. After buying $2$, we have $$2$ left. We cannot buy a third $$3$ box, so we opt for the $$2$ box instead (since it has a higher popsicles/dollar ratio than the $$1$ pack). We're now out of money. We bought $5+5+3=13$ popsicles, so the answer is $\boxed{\textbf{(D) }13}$.

Video Solution

https://youtu.be/str7kmcRMY8

https://youtu.be/9VGPpemq-qg

~savannahsolver

See Also

2017 AMC 10A (ProblemsAnswer KeyResources)
Preceded by
Problem 1
Followed by
Problem 3
1 2 3 4 5 6 7 8 9 10 11 12 13 14 15 16 17 18 19 20 21 22 23 24 25
All AMC 10 Problems and Solutions

The problems on this page are copyrighted by the Mathematical Association of America's American Mathematics Competitions. AMC logo.png